Quantcast
  • Register
PhysicsOverflow is a next-generation academic platform for physicists and astronomers, including a community peer review system and a postgraduate-level discussion forum analogous to MathOverflow.

Welcome to PhysicsOverflow! PhysicsOverflow is an open platform for community peer review and graduate-level Physics discussion.

Please help promote PhysicsOverflow ads elsewhere if you like it.

News

PO is now at the Physics Department of Bielefeld University!

New printer friendly PO pages!

Migration to Bielefeld University was successful!

Please vote for this year's PhysicsOverflow ads!

Please do help out in categorising submissions. Submit a paper to PhysicsOverflow!

... see more

Tools for paper authors

Submit paper
Claim Paper Authorship

Tools for SE users

Search User
Reclaim SE Account
Request Account Merger
Nativise imported posts
Claim post (deleted users)
Import SE post

Users whose questions have been imported from Physics Stack Exchange, Theoretical Physics Stack Exchange, or any other Stack Exchange site are kindly requested to reclaim their account and not to register as a new user.

Public \(\beta\) tools

Report a bug with a feature
Request a new functionality
404 page design
Send feedback

Attributions

(propose a free ad)

Site Statistics

205 submissions , 163 unreviewed
5,047 questions , 2,200 unanswered
5,345 answers , 22,709 comments
1,470 users with positive rep
816 active unimported users
More ...

  Why in SUGRA, N<= 8 if we restrict ourselves to particles of spin <=2?

+ 3 like - 0 dislike
912 views

As the title goes, in discussing SUGRA/SUSY, if we only consider particles of spin less than or equal to 2, then for SUGRA/SUSY, the maximal number of SUSY generators is N=8/N=4..Can anyone explain to me why it is the case or provide any source to read? Thanks.

asked Jul 6, 2014 in Theoretical Physics by wzr9999 (15 points) [ no revision ]

1 Answer

+ 4 like - 0 dislike

It is a standard fact which is explicitely or implicitely in every reference on supersymmetry. See for example the beginning of "Supersymmetry and supergravity" by Wess and Bagger.

The question is implicitely about spacetime dimension equal to 4. In this case, the smallest spinor representation is a Weyl spinor, with two complex components. To have $N$ supersymmetries means that we consider $N$ Weyl spinors $Q_{\alpha}^i$, $i=1,...,N$ ($\alpha=1,2$ is the Weyl spinor index), as fermionic generators of the supersymmetry algebra. The supersymmetry algebra is a super Lie algebra, extension of the Poincaré Lie algebra. Without central charges, it contains the relations

$\{ Q_\alpha^i, Q_\beta^j \} = 0$, $\{ \overline{Q}_{\dot{\alpha}}^i, \overline{Q}_{\dot{\beta}}^j\} =0$,

$\{ Q_\alpha^i, \overline{Q}_{\dot{\beta}}^j \} = 2 \sigma^\mu_{\alpha \dot{\beta}} P_\mu$

where $P_\mu$ is the 4-momentum operator and the  $\sigma^\mu_{\alpha \dot{\beta}}$ are the matrices giving the isomorphism between the vector representation and the tensor product of the left and right spinor representations of the Lorentz algebra ($\sigma^\mu = (id, - \sigma_i)$ with $\sigma_i$ the Pauli matrices). For a massless representation of the supersymmetry algebra, one can assume $P = (E,0,0,-E)$. In this case, the 2 by 2 matrix $2 \sigma^\mu_{\alpha \dot{\beta}} P_\mu$ has only one non-zero entry equal to $2E$. This implies that the supersymmetry algebra acting on this representation has $N$ fermionic creation operators. As a fermionic creation operator changes the spin of a state by 1/2, this means that the range of spin in a massless representation is equal to $N/2$. If $N>4$ then $N/2 >2$ so the representation contains a spin outside the interval [-1,1] : if all massless particles in the theory have spin $\leq 1$ then $N \leq 4$. If $N>8$ then $N/2 >4$ so the representation contains a spin outside the interval [-2,2]: if all massless particles in the theory have spin $\leq 2$ then $N \leq 8$. 

answered Jul 6, 2014 by 40227 (5,140 points) [ revision history ]
edited Jul 6, 2014 by 40227

Thank you for your detailed and kind reply!

Your answer

Please use answers only to (at least partly) answer questions. To comment, discuss, or ask for clarification, leave a comment instead.
To mask links under text, please type your text, highlight it, and click the "link" button. You can then enter your link URL.
Please consult the FAQ for as to how to format your post.
This is the answer box; if you want to write a comment instead, please use the 'add comment' button.
Live preview (may slow down editor)   Preview
Your name to display (optional):
Privacy: Your email address will only be used for sending these notifications.
Anti-spam verification:
If you are a human please identify the position of the character covered by the symbol $\varnothing$ in the following word:
p$\hbar$ysicsOver$\varnothing$low
Then drag the red bullet below over the corresponding character of our banner. When you drop it there, the bullet changes to green (on slow internet connections after a few seconds).
Please complete the anti-spam verification




user contributions licensed under cc by-sa 3.0 with attribution required

Your rights
...